Đến nội dung

Hình ảnh

Bất Đẳng Thức 2012


  • Please log in to reply
Chủ đề này có 79 trả lời

#41
morethanicansay545

morethanicansay545

    Binh nhì

  • Thành viên
  • 11 Bài viết
Bài 25: Cho $a,b,c>0$ thỏa $a+b+c=1$
$$P=\dfrac{a}{\sqrt{b+c}} +\dfrac{b}{\sqrt{a+c}} +\dfrac{c}{\sqrt{b+a}}$$

Pmin?????
MOD: Vui lòng gõ bài cho đầy đủ và vui lòng ghi số bài vào nhé.

Bài viết đã được chỉnh sửa nội dung bởi Ispectorgadget: 14-06-2012 - 19:44


#42
kainguyen

kainguyen

    Trung sĩ

  • Thành viên
  • 101 Bài viết

Bài 25: Cho $a,b,c>0$ thỏa $a+b+c=1$
$$P=\dfrac{a}{\sqrt{b+c}} +\dfrac{b}{\sqrt{a+c}} +\dfrac{c}{\sqrt{b+a}}$$

Pmin?????
MOD: Vui lòng gõ bài cho đầy đủ và vui lòng ghi số bài vào nhé.



Ta xét 2 biểu thức sau:

$A=\dfrac{a}{\sqrt{b+c}} +\dfrac{b}{\sqrt{a+c}} +\dfrac{c}{\sqrt{b+a}}$

và $B=a(b+c)+b(c+a)+c(a+b) =2(ab+bc+ca)$

Theo bđt Holder, ta có:

$A^2B\leq (a+b+c)^3$

Từ việc dự đoán GTNN của P, dẫn đến ta sẽ đi chứng minh:

$(a+b+c)^3\geq \frac{3}{2}B$

Thật vậy, ta có:

$(a+b+c)^3\geq \frac{3}{2}B$

$\Leftrightarrow (a+b+c)^3\geq 3(ab+bc+ca)$

$\Leftrightarrow (ab+bc+ca)\leq \frac{1}{3}$

đúng do $ab+bc+ca\leq \frac{(a+b+c)^2}{3}=\frac{1}{3}$

Vậy $Min P = \frac{\sqrt{6}}{2}$ tại $a=b=c=\frac{1}{3}$

Bài viết đã được chỉnh sửa nội dung bởi kainguyen: 14-06-2012 - 20:51


#43
kainguyen

kainguyen

    Trung sĩ

  • Thành viên
  • 101 Bài viết

Bài 24 Cho các số thực không âm thỏa mãn $a+b+c=1$. Tìm giá trị nhỏ nhất của biểu thức:
$$A=3(a^2b^2+b^2c^2+c^2a^2)+3(ab+bc+ca)+2\sqrt{a^2+b^2+c^2}$$



Ta có:

$3(a^2b^2+b^2c^2+c^2a^2)+3(ab+bc+ca)+2\sqrt{a^2+b^2+c^2}\geq (ab+bc+ca)^2+3(ab+bc+ca)+2\sqrt{1-2(ab+bc+ca)}$

Đặt: $ab+bc+ca=t$ Sau đó khảo sát hàm với $t \leq \frac{1}{3}$

#44
le_hoang1995

le_hoang1995

    Sĩ quan

  • Thành viên
  • 314 Bài viết

Ta có:

$3(a^2b^2+b^2c^2+c^2a^2)+3(ab+bc+ca)+2\sqrt{a^2+b^2+c^2}\geq (ab+bc+ca)^2+3(ab+bc+ca)+2\sqrt{1-2(ab+bc+ca)}$

Đặt: $ab+bc+ca=t$ Sau đó khảo sát hàm với $t \leq \frac{1}{3}$

Bạn ơi, bài này giá trị nhỏ nhất tại bộ $(0;0;1)$ cơ, và $min=2$

Đặt $ab+bc+ca=x \Rightarrow x\leq \frac{(a+b+c)^2}{3}=\frac{1}{3}\Rightarrow x\in[0,\frac{1}{3}] $
$$A=3(a^2b^2+b^2c^2+c^2a^2)+3(ab+bc+ca)+2\sqrt{a^2+b^2+c^2}\geq 3(ab+bc+ca)+2\sqrt{a^2+b^2+c^2}$$
$$=3(ab+bc+ca)+2\sqrt{(a+b+c)^2-2(ab+bc+ca)}=3x+2\sqrt{1-2x}$$
Xét hàm số $f(x)=3x+2\sqrt{1-2x}, \forall x\in[0,\frac{1}{3}] $
Đạo hàm $f'(x)=3-\frac{2}{\sqrt{1-2x}}$
$$f'(x)=0 \Leftrightarrow 3\sqrt{1-2x}=2\Leftrightarrow 9(1-2x)=4\Leftrightarrow x=\frac{5}{18}$$
Dựa vào bảng biến thiên, ta được $$f(x) \geq f(0), \forall x\in[0,\frac{1}{3}] $$
$$\Rightarrow A\geq f(x)\geq f(0)=2$$
Đẳng thức xảy ra khi $x=0$ hay hoán vị của bộ $(0,0,1)$
ĐPCM.

Bài viết đã được chỉnh sửa nội dung bởi le_hoang1995: 15-06-2012 - 09:57


#45
longqnh

longqnh

    Trung sĩ

  • Thành viên
  • 191 Bài viết
Bài 26: Cho $a,b,c>0$ thỏa mãn $a+b+c=3$ . Chứng minh rằng:

$\sum \frac{1}{a^{3}+2bc}\leq \frac{1}{abc}$

(đề thi thử ĐH lần 2 - THPT Gia Định - TPHCM)


SẼ KHÔNG BAO GIỜ BẾ TẮC NẾU TA CÒN CỐ GẮNG


#46
le_hoang1995

le_hoang1995

    Sĩ quan

  • Thành viên
  • 314 Bài viết

Bài 26: Cho $a,b,c>0$ thỏa mãn $a+b+c=3$ . Chứng minh rằng:

$\sum \frac{1}{a^{3}+2bc}\leq \frac{1}{abc}$

(đề thi thử ĐH lần 2 - THPT Gia Định - TPHCM)

Theo BĐT AM-GM ta có:
$$\sum \frac{1}{a^3+2bc}\leq \sum \frac{1}{3\sqrt[3]{a^3b^2c^2}}=\frac{\sqrt[3]{bc}+\sqrt[3]{ca}+\sqrt[3]{ab}}{3abc}$$
Lại theo BĐT Holder, ta có:$$\sqrt[3]{bc}+\sqrt[3]{ca}+\sqrt[3]{ab}\leq \sqrt[3]{(bc+ca+ab)(1+1+1)(1+1+1)}\leq \sqrt[3]{\frac{(a+b+c)^2.9}{3}}=\sqrt[3]{27}=3$$
Suy ra $$\sum \frac{1}{a^3+2bc}\leq \frac{\sqrt[3]{bc}+\sqrt[3]{ca}+\sqrt[3]{ab}}{3abc}\leq \frac{3}{3abc}=\frac{1}{abc}$$
Dấu bằng xảy ra khi $a=b=c=1$

#47
le_hoang1995

le_hoang1995

    Sĩ quan

  • Thành viên
  • 314 Bài viết
Bài 27 Cho $a,b,c>0$ thỏa mãn $a+b+c=1$. Tìm min:
$$A=\frac{a^2+b}{c+b}+\frac{b^2+c}{a+c}+\frac{c^2+a}{b+a}$$

#48
Jewellery

Jewellery

    Lính mới

  • Thành viên
  • 9 Bài viết
Bài 28: Cho $a,b,c$ thực dương thỏa $a^2+b^2+c^2+(a+b+c)^2\le 4$. Chứng minh rằng: $$\frac{ab+1}{(a+b)^2}+\frac{bc+1}{(b+c)^2}+\frac{ac+1}{(a+c)^2}\geq 3$$
USAMO 2011

#49
NGOCTIEN_A1_DQH

NGOCTIEN_A1_DQH

    Never Give Up

  • Thành viên
  • 625 Bài viết

Bài 28: Cho $a,b,c$ thực dương thỏa $a^2+b^2+c^2+(a+b+c)^2\le 4$. Chứng minh rằng: $$\frac{ab+1}{(a+b)^2}+\frac{bc+1}{(b+c)^2}+\frac{ac+1}{(a+c)^2}\geq 3$$
USAMO 2011


điều kiện của GT tương đương với:

$ (a+b)^2+(b+c)^2+(c+a)^2 \leq 4 $

đặt $ a+b=x, b+c=y, c+a=z $ thì $ a=\frac{x-y+z}{2}, b=\frac{x+y-z}{2}, c=\frac{-z+y+z}{2} $

thay vào ta được:

$ \frac{x^2-(y-z)^2+4}{4x^2}+\frac{y^2-(z-x)^2+4}{4y^2}+\frac{z^2-(x-y)^2+4}{4z^2} \geq 3 $

áp dụng điều kiện $ 4 \geq x^2+y^2+z^2 $ thì:


$VT \geq \frac{x^2+yz}{2x^2}+\frac{y^2+zx}{2y^2}+\frac{z^2+xy}{2z^2} \geq 3 $

cái này đúng theo AM-GM

Bài viết đã được chỉnh sửa nội dung bởi NGOCTIEN_A1_DQH: 22-06-2012 - 19:51

Em cắm hoa tươi đặt cạnh bàn

Mong rằng toán học bớt khô khan

Em ơi trong toán nhiều công thức

Cũng đẹp như hoa lại chẳng tàn

#50
Tham Lang

Tham Lang

    Thượng úy

  • Thành viên
  • 1149 Bài viết
Bài toán 29. [ Panagiote Ligouras, ML]
Bài này mình tìm trong 1 tuyển tập BĐT (ko có giải). Khá hay đó, mọi người thử chém nhé !
Cho các số thực dương $a,b,c$. Chứng minh rằng :
$$\sqrt{\dfrac{a+b}{c}}+\sqrt{\dfrac{b+c}{a}}+\sqrt{\dfrac{c+a}{b}}\ge 2\sqrt[6]{\dfrac{9(a+b+c)^4}{\sqrt[3]{abc}(a+b)(b+c)(c+a)}}$$

Off vĩnh viễn ! Không ngày trở lại.......


#51
viet 1846

viet 1846

    Gà con

  • Thành viên
  • 224 Bài viết

Bài 28: Cho $a,b,c$ thực dương thỏa $a^2+b^2+c^2+(a+b+c)^2\le 4$. Chứng minh rằng: $$\frac{ab+1}{(a+b)^2}+\frac{bc+1}{(b+c)^2}+\frac{ac+1}{(a+c)^2}\geq 3$$
USAMO 2011

ta có:

$ 2 \ge a^2+b^2+c^2+ab+bc+ca $ .
Ta Có : \[2.\sum_{cyc} \frac{1+ab}{(a+b)^2}=3+\sum_{cyc}\frac{(a+c)(b+c)}{(a+b)^2} \ge 6\]
Nên : \[ \sum_{cyc} \frac{1+ab}{(a+b)^2} \ge 3 \]

#52
Nguyễn Hoàng Lâm

Nguyễn Hoàng Lâm

    Sĩ quan

  • Thành viên
  • 312 Bài viết

Bài 27 Cho $a,b,c>0$ thỏa mãn $a+b+c=1$. Tìm min:
$A=\frac{a^2+b}{c+b}+\frac{b^2+c}{a+c}+\frac{c^2+a}{b+a}$

Xét $ P= \frac{a^2}{b+c}+ \frac{b^2}{c+a}+\frac{c^2}{a+b} $
Giả sử $ a\geq b\geq c $ Ta sẽ chứng minh :
$ P\geq \frac{c^2}{b+c}+\frac{a^2}{a+c}+\frac{b^2}{a+b} $
Một cách tổng quát xét bđt :
$ a_1.b_1+a_2b_2+a_3b_3 \geq a_1b_2+a_2b_3+a_3b_1 $ Với $\begin {align} &a_1 \geq a_2 \geq a_3 , & b_1 \geq b_2 \geq b_3 \end{align} $
$ \Leftrightarrow a_1(b_1-b_2)+a_2(b_2-b_3)+a_3(b_3-b_1) \geq 0 $
$ \Leftrightarrow VT = (b_1-b_2)(a_1-a_2)+ (b_1-b_3)(a_2-a_3) \geq 0 $ (Đúng theo giả thiết )
Từ đó Ta có :
Với $ a^2 \geq b^2 \geq c^2 $ và $ \frac{1}{b+c} \geq \frac{1}{c+a} \geq \frac{1}{a+b} $
Ta có:
$ P \geq \frac{c^2}{b+c}+\frac{a^2}{a+c}+\frac{b^2}{a+b} $
Đến đây ta có $ 2P \geq \frac{c^2+a^2}{b+c}+\frac{a^2+b^2}{a+c}+\frac{b^2+c^2}{a+b} \geq \frac{2ac}{b+c}+\frac{2ab}{a+c}+\frac{2bc}{a+b} $
$\Leftrightarrow P \geq \frac{ac}{b+c}+\frac{ab}{a+c}+\frac{bc}{a+b}$
Từ đó ta có :
$ A \geq \frac{ac+b}{c+b}+\frac{ba+c}{a+c}+\frac{cb+a}{b+a} = \frac{ac+b(a+b+c)}{c+b}+\frac{ba+c(a+b+c)}{a+c}+\frac{cb+a(a+b+c)}{b+a} =2(a+b+c)=2 $
Vậy $ A \geq 2 $
Đẳng thức xảy ra khi $ a=b=c = \frac{1}{3} $

Đôi khi ta mất niềm tin để rồi lại tin vào điều đó một cách mạnh mẽ hơn .


#53
le_hoang1995

le_hoang1995

    Sĩ quan

  • Thành viên
  • 314 Bài viết

Bài 27 Cho $a,b,c>0$ thỏa mãn $a+b+c=1$. Tìm min:
$$A=\frac{a^2+b}{c+b}+\frac{b^2+c}{a+c}+\frac{c^2+a}{b+a}$$

Bài này còn 1 cách nữa theo BDT AM-GM.
$$P+1=\frac{a^2+b}{c+b}+a+\frac{b^2+c}{a+c}+b+\frac{c^2+a}{b+a}+c$$
$$=\frac{a(a+b+c)+b}{b+c}+\frac{b(a+b+c)+c}{c+a}+\frac{c(a+b+c)+a}{a+b}$$
$$=\frac{a+b}{b+c}+\frac{b+c}{a+b}+\frac{c+a}{a+b}\geq 3$$
$$\Rightarrow A\geq 2$$
Dấu bằng xảy ra khi $a=b=c=\frac{1}{3}$

Bài viết đã được chỉnh sửa nội dung bởi le_hoang1995: 22-06-2012 - 23:23


#54
Ispectorgadget

Ispectorgadget

    Nothing

  • Quản lý Toán Phổ thông
  • 2946 Bài viết
Bài 30: Cho các số thực dương $a,b,c$ thỏa mãn $a+b+c=9$. Chứng minh rằng:$$log_3(a^2+18) + log_3(b^2+18) + log_3(c^2+18) \geq 9$$
Trích đề chọn HSG Chuyên TB

►|| The aim of life is self-development. To realize one's nature perfectly - that is what each of us is here for. ™ ♫


#55
Ispectorgadget

Ispectorgadget

    Nothing

  • Quản lý Toán Phổ thông
  • 2946 Bài viết
Bài 31: Cho $x,y,z$ thực thỏa $\frac{1}{4}\le x\le 1; y,z\geq 1;xyz=1$. Chứng minh rằng: $$\frac{1}{x+1}+\frac{1}{y+1}+\frac{1}{z+1}\geq \frac{22}{15}$$
Đề thi thử ĐH Trường THPT Thanh Thủy- Phú Thọ lần 2

Bài viết đã được chỉnh sửa nội dung bởi Ispectorgadget: 24-06-2012 - 18:16

►|| The aim of life is self-development. To realize one's nature perfectly - that is what each of us is here for. ™ ♫


#56
mathmath123

mathmath123

    Lính mới

  • Thành viên
  • 3 Bài viết
Bài 32:Cho a,b,c là các số thực dương.Chứng minh
$\frac{a^{4}+b^{4}+c^{4}}{ab+bc+ca}+\frac{3abc}{a+b+c}\geq \frac{2}{3}(a^{2}+b^{2}+c^{2})$

#57
viet 1846

viet 1846

    Gà con

  • Thành viên
  • 224 Bài viết

Bài 31: Cho $x,y,z$ thực thỏa $\frac{1}{4}\le x\lé; y,z\geq 1;xyz=1$. Chứng minh rằng: $$\frac{1}{x+1}+\frac{1}{y+1}+\frac{1}{z+1}\geq \frac{22}{15}$$
Đề thi thử ĐH Trường THPT Thanh Thủy- Phú Thọ lần 2


Sử dụng BĐT:

\[\frac{1}{{1 + y}} + \frac{1}{{1 + z}} \ge \frac{2}{{1 + \sqrt {yz} }} = \frac{2}{{1 + \frac{1}{{\sqrt x }}}} = \frac{{\sqrt x }}{{\sqrt x + 1}}\]

Bài toán trở về 1 biến:

Ngoài ra có thể chứng minh BĐT thông qua hàm Nêpe

Bài viết đã được chỉnh sửa nội dung bởi Hoàng Quốc việt: 24-06-2012 - 18:19


#58
viet 1846

viet 1846

    Gà con

  • Thành viên
  • 224 Bài viết

Bài 30: Cho các số thực dương $a,b,c$ thỏa mãn $a+b+c=9$. Chứng minh rằng:$$log_3(a^2+18) + log_3(b^2+18) + log_3(c^2+18) \geq 9$$
Trích đề chọn HSG Chuyên TB


Ta luôn có BĐT sau:

\[lo{g_3}({a^2} + 18) \ge 3 + \frac{2}{{9\ln 3}}\left( {a - 3} \right)\]

#59
NGOCTIEN_A1_DQH

NGOCTIEN_A1_DQH

    Never Give Up

  • Thành viên
  • 625 Bài viết

Bài 32:Cho a,b,c là các số thực dương.Chứng minh
$\frac{a^{4}+b^{4}+c^{4}}{ab+bc+ca}+\frac{3abc}{a+b+c}\geq \frac{2}{3}(a^{2}+b^{2}+c^{2})$

làm cách trâu bò:

chuẩn hóa cho $ a+b+c=3 $ thì bất đẳng thức trở thành:

$ a^4+b^4+c^4 +abc(ab+bc+ca) \geq \frac{2}{3}(a^2+b^2+c^2)(ab+bc+ca) $

đặt $ 3=a+b+c=p, ab+bc+ca=q, abc=r $ và chú ý các đẳng thức, bất đẳng thức sau:

$ a^4+b^4+c^4=p^4-4p^2q+2q^2+4pr=81-36q+2q^2+12r $
$ a^2+b^2+c^2=p^2-2q=9-2q $
$ r \geq \frac{p(4q-p^2)}{9}=\frac{4p-9}{3} $ (bất đẳng thức schur)
thay vào bdt cần chứng minh ta được:

$ BDT \Leftrightarrow \frac{10}{3}q^2-42q+qr+12r+81 \geq 0 $ (1)

áp dụng bất đẳng thức schur ta có:

$ VT(1) \geq \frac{10}{3}q^2-42q+\frac{q(4q-9)}{3}+4(4q-9)+81 $

ta sẽ chứng minh $ \frac{10}{3}q^2-42q+\frac{q(4q-9)}{3}+4(4q-9)+81 \geq 0 $

$ \Leftrightarrow 14q^2-87q+135 \geq 0 $

$ \Leftrightarrow (q-3)(q-\frac{45}{14}) \geq 0 $ ( luôn đúng vì $ q \leq 3 $)

vậy bdt được chứng minh
Em cắm hoa tươi đặt cạnh bàn

Mong rằng toán học bớt khô khan

Em ơi trong toán nhiều công thức

Cũng đẹp như hoa lại chẳng tàn

#60
mathmath123

mathmath123

    Lính mới

  • Thành viên
  • 3 Bài viết
Bài này không cần chuẩn hóa đâu bạn à :)
__
Bạn làm cụ thể ra luôn đi nhé!

Bài viết đã được chỉnh sửa nội dung bởi Ispectorgadget: 24-06-2012 - 20:57





0 người đang xem chủ đề

0 thành viên, 0 khách, 0 thành viên ẩn danh